若.k45k9是能被3整除的五位数,则k的可能取值有 ______个;这样的五位数中能被9整除的是 ______.

chenzhijia2022-10-04 11:39:541条回答

已提交,审核后显示!提交回复

共1条回复
饭米索儿 共回答了25个问题 | 采纳率88%
解题思路:由已知,若k45k9能被3整除,则(k+4+5+k+9)是3的倍数,即2k+18是3的倍数,由此可求出k,然后用求得k的数组成的五位数的5个数的和那个是9的倍数即得答案.

已知,五位数k45k9能被3整除,
所以(k+4+5+k+9)是3的倍数,
即2k+18是3的倍数,
18是3的倍数,
则2k是3的倍数,
3,6,9,12,15,18…是3 的倍数,
又K是1、2、3、4、5、6、7、8、9,其中的数,
如果k=1,2,4,5,7,8时,2k不是3的倍数,
当k=3,6,9时,
2k是3的倍数,
所以k=3或6或9,
得到3个五位数即34539,64569,94599,
而这三个五位数中只有94599的5个数的和是9的倍数.
所以这样的五位数中能被9整除的是94599.
故答案分别为:3,94599.

点评:
本题考点: 数的整除性;数的十进制.

考点点评: 此题是考查数的整除性问题,解答的关键是这个五位数能被3或9整除,则有它们5个数的和是3或9的倍数.

1年前

相关推荐

同时抛掷两枚质地均匀的骰子,骰子的六个面分别刻有1到6的点数,朝上的面的点数中,一个点数能被另一个点数整除的概率是(
同时抛掷两枚质地均匀的骰子,骰子的六个面分别刻有1到6的点数,朝上的面的点数中,一个点数能被另一个点数整除的概率是(  )
A. [7/18]
B. [3/4]
C. [11/18]
D. [23/36]
一支红玫1年前3
ainihm 共回答了19个问题 | 采纳率100%
解题思路:列举出所有情况,看朝上的面的点数中,一个点数能被另一个点数整除的情况数占总情况的多少即可.

可用列表法表示出同时抛掷两枚质地均匀的骰子的结果,发现共有36种可能,由于没有顺序,因此发现,在这36种结果中,一个点数能被另一个点数整除的情况出现了22次.
∴一个点数能被另一个点数整除的概率是[22/36]=[11/18].

(1,6) (2,6) (3,6) (4,6) (5,6) (6,6)
(1,5) (2,5) (3,5) (4,5) (5,5) (6,5)
(1,4) (2,4) (3,4) (4,4) (5,4) (6,4)
(1,3) (2,3) (3,3) (4,3) (5,3) (6,3)
(1,2) (2,2) (3,2) (4,2) (5,2) (6,2)
(1,1) (2,1) (3,1) (4,1) (5,1) (6,1)故选C.

点评:
本题考点: 列表法与树状图法.

考点点评: 本题考查的是对概率的理解和简单的计算;采用列举法解题的关键是找到所有存在的情况.用到的知识点为:概率=所求情况数与总情况数之比.

设某年为s,判断其是否为闰年.闰年的判断方法:1) 如果年份能被400整除,该年份为闰年.2) 如果年份能背4整除而且不
设某年为s,判断其是否为闰年.
闰年的判断方法:
1) 如果年份能被400整除,该年份为闰年.
2) 如果年份能背4整除而且不能被100整除,该年份为闰年.
则判断闰年的条件可以写成以下计算机表达式:
(S Mod 400=0) _______(s Mod 4=0 And s Mod 1000)
ljxliu771年前1
DaisyBIT 共回答了24个问题 | 采纳率100%
填写.OR
a能整除23 a是
jackbaby1234561年前5
zhen6671207 共回答了15个问题 | 采纳率93.3%
a是1
祝你开心!
某个七位数1993( )( )( )能被2、3、4、5、6、7、8、9都整除、那么它的最后
某个七位数1993( )( )( )能被2、3、4、5、6、7、8、9都整除、那么它的最后
某个七位数1993( )( )( )能被2、3、4、5、6、7、8、9都整除、那么它的最后三个数字组成的三位数是多少?
dsdsdsdsds32321年前1
821291 共回答了20个问题 | 采纳率60%
它的最后三个数字组成的三位数是3、2、0~即1993320
12,15,20都能整除同一个自然数,这个数最小是多少?
逍遥的懒猪1年前6
smufcuwxf 共回答了18个问题 | 采纳率94.4%
解题思路:12,15,20都能整除同一个自然数,这个自然数是它们的最小公倍数,即应该是60.

把12、15和20分解质因数:
12=2×2×3;
15=3×5;
20=2×2×5;
12、15和20的最小公倍数是:2×2×3×5=60;
答:这个数最小是60.

点评:
本题考点: 求几个数的最大公因数的方法.

考点点评: 此题属于最小公倍数 问题,按照求三个数的最小公倍的方法,求出它们的最小公倍数问题即可解决.

从0,1,2,3,5,7中选出三个不重复的数字,组成一个既能被5整除,又能被3整除的最大三位数,这个三位数是______
从0,1,2,3,5,7中选出三个不重复的数字,组成一个既能被5整除,又能被3整除的最大三位数,这个三位数是______.
阳光大松1年前1
2ifpp 共回答了13个问题 | 采纳率84.6%
符合条件的三位数有:120、210、102、150、510、105、270、720、207、705、750、570、135、315、513、
375、735、573,
其中最大的三位数是750;
故答案为:750.
19乘以一个数积是质数;乘以另一个数积是合数,并且能被1,2,3,4,…等自然数整除.问这两个数(不能是分数或小数)分别
19乘以一个数积是质数;乘以另一个数积是合数,并且能被1,2,3,4,…等自然数整除.问这两个数(不能是分数或小数)分别是什么数?
盘锦拉拉1年前1
钻石hh 共回答了18个问题 | 采纳率88.9%
解题思路:因为19乘一个数积是质数,这个数只能是1;乘以另一个数积是合数,并能被1,2,3,4,…自然数整除,另一个数能被1,2,3,4整除,这个数是12,所以这两个数(不能是分数或小数)分别是1、12.

因为19乘一个数积是质数,这个数只能是1;
乘以另一个数积是合数,并能被1,2,3,4,…自然数整除,另一个数能被1,2,3,4整除,这个数是12,
所以这两个数(不能是分数或小数)分别是1、12.
答:这两个数(不能是分数或小数)分别是1、12.

点评:
本题考点: 整除性质.

考点点评: 本题主要是根据整数和合数的意义及整除的性质解决问题.

1:从1到10这10个数中任取不同的3个数,相加后能被3整除的概率是?
1:从1到10这10个数中任取不同的3个数,相加后能被3整除的概率是?
2:将5本不同的书全发给4名同学,每名同学至少有一本书的概率是?
3:tan20°+tan40°+√3tan20°tan40°=?
(sin65°+sin15°sin10°)/sin25°-cos15°cos80°=?
4:用^表示乘方,用log(a)(b)表示以a为底,b的对数 :
log(2)(cos(π/9))+log(2)(cos(2π/9))+log(2)(cos(4π/9))=?
5:在△ABC中,面积S=a^2-(b-c)^2,则cosA=?
6:在等差数列(an)中ap=q,aq=p(p不等于q),则a(p+q)的值是?
7:若关于x的不等式组x^2-x-2>0 的整数解得集合为(-2),求实数k的取值范围.
2x^2+(2k+5)x+5k=2 B:(a+b+c)^2>=3
C:1/a+1/b+1/c>=2√3 D:a+b+c
wangkaiwu19981年前1
韬声依旧1979 共回答了17个问题 | 采纳率94.1%
1.首先,1到10可以有120种组合的方式,其中满足被3整除的有{1,2,3}、{1,2,6}、{1,2,9}、{1,3,5}、{1,3,8}、{1,4,7}、{1,4,10}、{1,5,6}、{1,5,9}、{1,6,8}、{1,7,10}、{1,8,9}、{2,3,4}、{2,3,7}、{2,3,10}、{2,4,6}、...
有n个大于10的连续正整数,它们的各位数码之和都不能被7整除.问n的最大值是多少
有n个大于10的连续正整数,它们的各位数码之和都不能被7整除.问n的最大值是多少
DeadE1年前0
共回答了个问题 | 采纳率
写一个大于100被3整除的整数,求这个数每一个位上的数的立方,将所得数重复操作.一直这样,发现什么
爱-情1年前1
zyzy4000 共回答了13个问题 | 采纳率92.3%
随便挑一个符合要求的数,重复上述操作,会发现不管什么数,这样算下去都会得到153这个数,你去算算吧……我们老师说的……
在1,2,3……10000这1000个整数中,计算能被2整除但不能被三整除的数字有几个 能被2整除或不能被3整除的数的
在1,2,3……10000这1000个整数中,计算能被2整除但不能被三整除的数字有几个 能被2整除或不能被3整除的数的
在1,2,3……10000这1000个整数中,计算能被2整除但不能被三整除的数字有几个 能被2整除或不能被3整除的数有几个
connor121年前1
gdhhjh 共回答了27个问题 | 采纳率100%
能被2整除的有:1000/2=500个
其中能被6整除的有:[500/6]=83个
能被2整除但不能被三整除的数字有:500-83=417个
证明5的8次方能被20至30的两个数整除
食指上的烟味1年前2
tuuo7ey 共回答了22个问题 | 采纳率100%
题目是不是有问题啊
以前我看见的是“证明5^8-1能被20至30的两个数整除”
数学好的进来看看.本人数学白痴(1)设n为整数,利用因式分解说明(2n+1)²-25能被4整除.(2)已知a、b、c均为
数学好的进来看看.本人数学白痴
(1)设n为整数,利用因式分解说明(2n+1)²-25能被4整除.
(2)已知a、b、c均为正数,且2a²b+2a²c-2ab²-2abc=0,求证:a=b
(3)将若干只鸡放入若干个笼子中,若每笼放4鸡,则有1鸡无笼可放,若每笼放5鸡,则有一笼无鸡可放,问有几只鸡,几个笼子?
虽然不是很高,
夜灵儿5201年前1
rrggg发起人 共回答了30个问题 | 采纳率90%
第一题,把它化简开来得到4n²+4n-24,可以提取公因式4啊,所以肯定能被4整除
第二题,先把2消掉,然后把有C的提取公因式,得到
a²b-ab²+ac(a-b)=0
前面也提取公因式得到
ab(a-b)+ac(a-b)=0
消去a得到
(b+c)×(a-b)=0
都是正数所以b+c≠0
所以a-b=0,即得到a=b
第三题,可以设x个笼子
4x+1=5(x-1)
解得 x=6
即为六个笼子,
所以有6×4+1=25只鸡
请用数学归纳法证明.大于1的整数都能被some primes(素数)整除~用英文回答再加20分~
好奇囡囡1年前1
乳猪 共回答了21个问题 | 采纳率76.2%
这个真不会,是不是楼主的题目出错了?根据质数和合数的定义可以直接得到命题结论啊,不需要数学归纳法.
2009--2010学年度第二学期六年级数学试卷(数的整除)
2009--2010学年度第二学期六年级数学试卷(数的整除)
一个数只有1和它本身两个因数,这样的数叫做().
a,b是两个自然数,且a÷b=12,则a和b的最大公因数是()
冰点甲醇1年前1
偶然在转角 共回答了25个问题 | 采纳率92%
素数(质数)
在10~100这些自然数中,那些能被3整除的数有几个?被3除余数是1,被3除余数是2的数有几个?
李华19821年前2
qazsew123 共回答了13个问题 | 采纳率92.3%
9/3=3
100/3=33余1
10—100共有91个数
所以能被3整除的数有90-(33-3)=60
剩下的数不是被3除余1就是余2
我们把每三个数看成是一组的话
10、11、12,13、14、15,.97、98、99,一共是30组 还剩最后一个数100
每组数中分别有一个数被3整除,一个数余1,一个数余2
所以被3整除余1的有30+1=31
被3整除余2的有30
质地均匀的正四面体骰子的四个面上分别写有数字:2,3,4,5.投掷这个正四面体两次,则第一次底面上的数字能够整除第二次底
质地均匀的正四面体骰子的四个面上分别写有数字:2,3,4,5.投掷这个正四面体两次,则第一次底面上的数字能够整除第二次底面上的数字的概率是______.
wwwww300001年前1
苍穹星伴月 共回答了18个问题 | 采纳率100%
由树状图


可知共有4×4=16种可能,第一次底面上的数字能够整除第二次底面上的数字的有5种,所以概率是
5
16 .
被3整除的数为___,被3除余1的数为___.(用含x的代数式表示)
jacqui13045461年前2
laoqiu2520 共回答了13个问题 | 采纳率84.6%
被3整除的数为3x, 被3除余1的数为3x+1
设f(x)、g(x)为整系数多项式,且g(x)首相系数为1,证明g(x)整除f(x)的充分必要条件是存在无穷多整数n使g
设f(x)、g(x)为整系数多项式,且g(x)首相系数为1,证明g(x)整除f(x)的充分必要条件是存在无穷多整数n使g(n)整除f(n)
dcyc1年前1
lizongmu 共回答了19个问题 | 采纳率78.9%
(1)
g(x)|f(x),那么对于任意的n都有,g(n)|f(n)
(2)
要证明多项式整除,一般采取验证它的余式为0.
要想有余式,那么要求f(x)的次数比g(x)要至少一样大.
下面证明.
既然有无穷多个整数都满足g(n)|f(n),根据皮亚诺公理,
那么一定存在充分大的整数满足g(n)|f(n).
假若def(g(x))>def(f(x)),那么
可以取到足够大的整数,使得g(n)>f(n),与已知条件矛盾.
于是证明了def(g(x))≤def(f(x))
那么可以按余式形式,设
f(x)=p(x)·g(x)+r(x),其中def(r(x))≤def(g(x))
那么显然是有无穷多个n,使得
f(n)=p(n)·g(n)+r(n),
注意到,因为n是数字,
因而上面的式子不是多项式,是数字的带余数除法,那么我们可以作算术除法:
f(n)/g(n)-r(n)/g(n)=p(n)
注意到,p(n)一定是整数.
既然g(n)|f(n),
那么会有r(n)/g(n),并且存在无穷多个n都满足.
由def(r(x))≤def(g(x)),
那么对于充分大的n,一定存在g(n)>r(n)
只能r(x)=0
证明完毕.
【经济数学团队为你解答!】
数学题(关于数的整除)将既能被5除又能被7除的自然数从35起,按从小到大排成一行,问:这列数中1993个数的和被11除的
数学题(关于数的整除)
将既能被5除又能被7除的自然数从35起,按从小到大排成一行,问:这列数中1993个数的和被11除的余数是多少?
jungleyu1年前1
13紫川 共回答了28个问题 | 采纳率92.9%
先算出最后一项的数
(1993-1)*35+35=69755
再算出和
(35+69755)*1993/2=69545735
69545735除以11的余数为
(5+7+4+9)-(3+5+5+6)=6
6即为余数
从0,1,2,3,5,7中选出三个不重复的数字,组成一个既能被5整除,又能被3整除的最大三位数,这个三位数是______
从0,1,2,3,5,7中选出三个不重复的数字,组成一个既能被5整除,又能被3整除的最大三位数,这个三位数是______.
keven_20041年前1
fywind 共回答了16个问题 | 采纳率100%
解题思路:既能被5整除,又能被3整除的数必须具备:个位上的数是0或5,各个数位上的数的和能够被3整除,符合条件的三位数有120、210、102、150、510、105、270、720、207、705、750、570、135、315、513、375、735、573,其中最大的三位数是750.

符合条件的三位数有:120、210、102、150、510、105、270、720、207、705、750、570、135、315、513、
375、735、573,
其中最大的三位数是750;
故答案为:750.

点评:
本题考点: 整除的性质及应用.

考点点评: 此题属于考查能3、5整除的数的特征,记住特征,灵活解答.

有一个41位数前20位是5后20位是9中间是什么数字能被7整除?
peterstone1131年前2
007小子 共回答了21个问题 | 采纳率95.2%
前20个5每6个5都能被7整除,所以前18个5能被7整除.
后20个9中每6个9都能被7整除,所以后18个9能被7整除.
现在只要中间的55x99能被7整除就可以了.现在来找x的取值
用试值法:
x=0,55099/7不能整除
x=1,55199/7不能整除
x=2,55299/7不能整除
x=3,55399/7不能整除
x=4,55499/7不能整除
x=5,55599/7不能整除
x=6,55699/7能整除,所以x=6可取
x=7,55799/7不能整除
x=8,55899/7不能整除
x=9,55999/7不能整除
故中间数是6
求两道数论整除证明 如图 第二个 m>n第二题这样的话m,n就一直差1啊 不能满足任意m>n吧
求两道数论整除证明 如图


第二个 m>n第二题这样的话m,n就一直差1啊 不能满足任意m>n吧
wei520991年前0
共回答了个问题 | 采纳率
试说明:对于任何正整数n,2的n+4次方-2^n必能被3整除
moseny1年前2
intorain 共回答了26个问题 | 采纳率100%
2的n+4次方-2^n=2^ n(2^4-1)=2^n*15

[2^(n+4)-2^2]/3=15*2^n/3=5*2^n为整数,故
对于任何正整数n,2的n+4次方-2^n必能被3整除
甲数和乙数的最大公约数是1,丙数能整除乙数,那么,甲、乙、丙三个数的最小公倍数是(  )
甲数和乙数的最大公约数是1,丙数能整除乙数,那么,甲、乙、丙三个数的最小公倍数是(  )
A. 甲数
B. 乙数
C. 甲数×乙数
D. 乙数×丙数
秦人66661年前1
wrlzrm 共回答了23个问题 | 采纳率95.7%
解题思路:由题意知:丙数能整除乙数,那么乙数和丙数的最小公倍数就是乙数;又因为甲乙两个数的最大公约数是1,即甲数和乙数是互质的,最小公倍数就是他们的乘积;进而得出结论.

由分析知:乙数和丙数的最小公倍数就是乙数,甲乙两个数的最小公倍数就是它们的乘积;
所以甲乙丙三个数的最小公倍数是甲数和乙数的乘积,即甲数×乙数.
故选:C.

点评:
本题考点: 求几个数的最小公倍数的方法;求几个数的最大公因数的方法.

考点点评: 此题属于易错题,解答此题应结合题意,根据求几个数的最小公倍数的方法进行解答.

57、96、和148被某个自然数N整除,余数相同,N的最大值是多少
fongson1年前2
sk8erboi 共回答了20个问题 | 采纳率90%
57=na+r
96=nb+r
148=nc+r
相互作差
39=(a-b)n=13*3 52=(c-b)n=13*4 91=(c-a)n=13*7
39=(a-b)n=13*3显然(a-b) n整数=13或3 所以n=13最大
一个数用它去除112.136都正好整除,这个数最大是几?
张舒乔1年前3
呢喃囡囡 共回答了16个问题 | 采纳率93.8%
注意题中的条件是去除,而不是去除以,所以是去找这两个数的最大公约数
112=2*2*2*2*7
136=2*2*2*17
所以它们的最大公约数是8
也就是这个数最大是8
欢迎追问
数的整除 概念用通俗的语言说
joyfingers1年前2
光光ss 共回答了17个问题 | 采纳率76.5%
两个整数相除,除数不为0,又没有余数.
圆的一周定义它为360度,而不是400度,是不是因为一天24小时,正好能被360整除,而400却不能
错落ll1年前1
太寒心 共回答了14个问题 | 采纳率92.9%
古时候,人们观察天体运行的变化,产生自然现象的变化.其中似乎关联到一个循环的周期现象;并且和圆有相当的关系.观察到一年有四季的变化,每一季潮汐的变化中发现月亮在一季中有3次的循环,也就是一季有3个月,在每个月里,太阳的上升与落下,大约有30次 的循环,所以一个月有30天;一年就有360天.这或许跟圆的360度有关系.
如果两个数之和是64,两数的积可以整除4875,那么这两数之差是______.
面团七七1年前1
yyyy 共回答了17个问题 | 采纳率100%
解题思路:根据两数之积能整除4875,把4875分解质因数,再根据两数之和为64进行组合,推得这两数为39和25,从而求出它们的差.

4875=3×5×5×5×13
=(3×13)×(5×5)×5,
=(39×25)×5,
且39+25=64,
所以这两数为39和25.
39-25=14.
故答案为:14.

点评:
本题考点: 合数分解质因数.

考点点评: 考查了合数分解质因数,解题的根据是将4875分解为(39×25)×5,结合两个数之和是64,依此确定这两数为39和25.

证明在任意给出的n+2(n>=2)个整数中必有两个数,它们的差能被n整除
证明在任意给出的n+2(n>=2)个整数中必有两个数,它们的差能被n整除
用鸽笼原证明
落樱满肩1年前1
dvfqjs 共回答了17个问题 | 采纳率88.2%
证明:
设任意n+2个整数为m,m+1,m+2,...,m+n+1(m为任意整数),
由于[(m+n+1)-(m+1)]/n=n/n=1,
即证.
甲数比乙数大5,乙数比丙数也大5,试说明三数之和 三数之积都能被3整除
lyh59031年前5
hygxrb 共回答了20个问题 | 采纳率95%
甲数比乙数大5,乙数比丙数也大5,甲比丙大10.
这三个数之和可以表示为:(乙+5)+乙+(乙-5)=乙×3 能被3整除
这三个数之积可以表示为:(丙+10)(丙+5)丙
若丙是3的倍数,则(丙+5)÷3余2,(丙+10)÷3余1;即这三个因数分别除以3
所得的结果分别是余1、余2和不余;也就是说:这三个因数中有一个因数一定是3的倍数.
所以三数之积能被3整除.
在70以内能被3整除.蛋不能被7整除的所有正数的和是
季仪1年前2
zhouyanxu 共回答了16个问题 | 采纳率93.8%
70以内能被3整除的数是3,6,9,...,69,其中能被7整除的数一定能被21整除,所以70以内能被3整除且能被7整除的有21,42,63三个数,
70以内能被3整除的数的和是(3+69)*21/2=756
再减去能被7整除的三个数的和就是756-(21+42+63)=630
下面那种说法是对的 A奇数不能被2整除B偶数的个位一定是2C5的倍数个位上一定是5D2个奇数的和一定是奇数
下面那种说法是对的 A奇数不能被2整除B偶数的个位一定是2C5的倍数个位上一定是5D2个奇数的和一定是奇数
下面那种说法是对的 A奇数不能被2整除 B偶数的个位一定是2 C5的倍数个位上一定是5 D2个奇数的和一定是奇数
mcjayr1年前4
goooogol 共回答了18个问题 | 采纳率88.9%
A奇数不能被2整除 对
B偶数的个位一定是2 还可能是0
C5的倍数个位上一定是5 还可能是偶数0
D2个奇数的和一定是奇数 一定是偶数
选A
1.用某数去除56,108和162,都不能整除.又知他们向除后三个余数之和正好是20.这个数是多少?
1.用某数去除56,108和162,都不能整除.又知他们向除后三个余数之和正好是20.这个数是多少?
2.有一个四位数,当它被131除时,余数是112;当它被132除时,余数是98.这个四位数是多少?
3.用一个奇数去除288和214,所得的余数都是29,这个奇数是多少?
好的+++++++啊
呵呵o(∩_∩)o...
写清楚一点啊
1203061年前3
宝鸡03 共回答了21个问题 | 采纳率81%
第一题:
假设该数除56的余数为x
则该数除112的余数为该数除2x后得的余数
则该数除108和162得的余数同理为其除2x-4,3x-6后得的余数
6x-10=20,x=5
求得该数为17 或者51
该解法得到的答案正确,但是逻辑上有一步不严谨,望高人指正.
第二题:
因为第一次除131余数为112,第二次除132余数为98
132比131只大1,所以得知商相等,设为x
则131x+112=132x+98
x=14
代回,14*131+112=1946
第三题:
一个奇数去除288和214余数都为29
则该数可以被259=288-29和185=214-29整除
则该数分解259和185分别为259=7*37,185=5*37
则该奇数为37
试写出5个自然数,使得其中任意两个数中较大的一个数可以被这两个数的差整除.
qwmbeyond1年前3
悲伤cute 共回答了12个问题 | 采纳率66.7%
解题思路:先写出满足要求的三个数2,3,4,然后分别用a写出满足条件的四个数,用b写出满足条件的五个数,从而利用正处的性质即可得出答案.

1680,1692,1694,1695,1696为满足条件的5个数(答案不唯一).
以上5个数可用以下步骤找出:
第一步:2,3,4为满足要求的三个数;
第二步:设a,a+2,a+3,a+4为满足条件的四个数,则a可被2,3,4整除,
取a=12,得满足条件的四个数为12,14,15,16;
第三步:设b,b+12,b+14,b+15,b+16为满足条件的五个数,
取12,14,15,16的最小公倍数为b,即b=1680,
得满足条件的五个数1680,1692,1694,1695,1696.

点评:
本题考点: 数的整除性.

考点点评: 本题考查数的整除性的知识,难度较大,解决本题的关键是先找出简单的符合条件的少量数,以此为基础是按照步骤依次进行解答.

2的96次方减1一定被60至70之间的两个整数整除,你知道这两个整数是多少?
有熊舞于野1年前2
忘情太平洋 共回答了14个问题 | 采纳率78.6%
2^96-1=(2^48+1)(2^48-1)=(2^48+1)(2^24+1)(2^24-1)=…… =(2^48+1)(2^24+1)(2^12+1)(2^6+1)(2^6-1) 最后两项为65和63
2006平方+2006能被2006整除吗?能被2007整除吗?为什么?
ay42861年前6
Deena 共回答了18个问题 | 采纳率77.8%
都能.2006的平方加上2006
相当于 2006×2006+2006=2006×(2006+1)=2006×2007
所以能被2006和2007整除.
在正整数集合Z^+中,设xRy意味着x-y能被7整除.试证明R是一个等价关系并写出其等价类.
在正整数集合Z^+中,设xRy意味着x-y能被7整除.试证明R是一个等价关系并写出其等价类.
RT
sweetice_20061年前1
晏妍 共回答了18个问题 | 采纳率94.4%
x-x=0能被7整除如果x-y能被7整除,y-x=-(x-y)也能被7整除如果x-y能被7整除,y-z能被7整除,那么x-z=(x-y)+(y-z)也能被7整除所以R是等价关系等价类有7个:{x:x=0(mod 7)},{x:x=1(mod 7)},{x:x=2(mod 7)},{x:x=3(mod 7)}{...
若二次多项式3x^2+6kx-9k^2能被x-1整除,则k的值为()
wozj1年前1
gzvibom 共回答了26个问题 | 采纳率88.5%
设另一个因式是A
则3x²+6kx-9k²=A(x-1)
x=1时,x-1=0
所以右边A(k-1)=0
所以左边也等于0
所以x=1
3x²+6kx-9k²=3+6k-9k²=0
3k²-2k-1=0
(3k+1)(k-1)=0
k=-1/3,k=1
若2x2次+3x-9可被x+k整除,求常数k的值
richiempaul1年前1
waynewei 共回答了18个问题 | 采纳率100%
2x2+3x-9=2(x+3)(x-3/2)
x+k整除2x2+3x-9
k=3或k=3/2
1到3009含1和300)这300个数字中,不能被3整除的数字和是多少
1到3009含1和300)这300个数字中,不能被3整除的数字和是多少
如题~要有式子和计算过程!
ninzilan1年前1
123456789asdfghj 共回答了13个问题 | 采纳率92.3%
1+4+7+.298
2+5+8+.299
3+6+9+.300
各100个数
要求就是求前两行的和
第3行-第2行=第2行-第1行=100
3行总和=301*150=45150
假设第一行为x
3x+100+200=45150
x=14950
所以第1行+第2行=2x+100=30000
个位数字是6而又能被3整除的五位数有几个
冷__1年前3
Richer 共回答了17个问题 | 采纳率94.1%
五位数中最小能被3整除的是10002
10002÷3=3334
五位数中最大能被3整除的是99999
99999÷3=33333
个位数字是6的五位数,除以3,得到的商,个位数字必是2
所以,题目即要求3334~33333中个位是2的数有几个
从3334往上排,每10个数中有一个个位是2
3334~3340中个位不含2,33331~33333中有一个个位含2
3340~33330中:(33330-3340)÷10=2999,有2999个数尾数含2
2999+1=3000
所以,符合题目要求的数有3000个.
一个数能被2,4,8分别整除,且除以3余1,除以5余3,除以6余4,除以7余4.求符号条件的最小的数
moongg131年前3
poohshan 共回答了19个问题 | 采纳率94.7%
根据条件可知,这个数能被8整除,加上2之后被30整除.由此可知这个数加上2之前个位是8,由此可列出被8整除的、个位是8的数:
8 48 88 128 168 208 248 288 328……
其中88、208、328符合“加上2之后被30整除”
88/7=12……4
208/7=29……5
328/7=46……6
由此可知88是符合条件的最小的数
求自然数1-100中,不能被3整除的所有数的和
nordoffhall1年前6
dragon840722 共回答了17个问题 | 采纳率82.4%
先不考虑100.去掉3的倍数后剩下所有数的和为1+2+4+5+.94+95+97+98
一头一尾的和为99;这样的和有33组,其和为99X33=3267;再加100,自然数1-100中,不能被3整除的所有数的和为3367.
试找出连续的21个自然数,使得其中每一个数都能被2至13中的某一个自然数整除
ru89861年前1
红颜如玉碎 共回答了25个问题 | 采纳率92%
结论可以更强,加强为:使得其中每一个数都能被2至11中的某一个自然数整除.
首先构成11的阶乘,即11!,则
11!-10, 11!-9, 11!-8, 11!-7,.11!, 11!+1, ...11!+10 这十个数就符合条件
491至少要增加多少才能同时被二和三整除 少要减少多少能被三和五同时整除
491至少要增加多少才能同时被二和三整除 少要减少多少能被三和五同时整除
491至少要增加多少才能同时被二和三整除 少要减少多少能被三和五同时整除 求
laoshimen1年前1
我是年年 共回答了21个问题 | 采纳率81%
491至少要增加多少才能同时被二和三整除
2和3的公倍数接近491的只有492,至少增加1
证明从1-200个数中取100个整数,其中之一小于16,那么必有两个数,一个能被另一个整除.利用鸽巢原理
梦寻梨落1年前2
猪的逻辑 共回答了15个问题 | 采纳率86.7%
有n+1只鸽子进入n个笼子,那么必然有至少两只鸽子在同一个笼子中.
q1,q2,q3,……,qn是n个正整数,则 q1+q2+q3+……+qn-n+1 个物体放入n个盒子中,那么,或者第一个盒子中至少有q1个物,或者第二个盒子中至少 有q2个物体,或者第三个盒子中至少有q3个物体,……,或者第n个盒子中至少有qn个物 体.我们通常提到的鸽巢原理的定义是这种严谨的定义的一个特例,也就是设qx=2(其中x为1,2,3,……,n),那么上面定义中的q1+q2+q3+……+qn-n+1就简化为n+1
如果一个数可以被一个自然数整除,为什么其各位数字之和要能被该数整除
wugangning1年前3
hnhklyh 共回答了21个问题 | 采纳率95.2%
3 与 9 符合此规律,其它的不行
如 28 能被 7 整除 ,而 2+8=10 不能被 7 整除